Keresés


Toplista

Toplista
  • betöltés...

Magántanár kereső

Ha szívesen korrepetálnál, hozd létre magántanár profilodat itt.
Ha diák vagy és korrepetálásra van szükséged, akkor regisztrálj be és írd meg itt, hogy milyen tantárgyban!

Fizika

872
Egy 60°-os 3 méter magas lejtő tetejéről egyenletesen lecsúszik egy test. Milyen magasra jut a lejtőn ugyanez a test, ha a lejtő alján meglökjük 5 m/s sebességgel?
Jelenleg 1 felhasználó nézi ezt a kérdést.
0
Középiskola / Fizika

Válaszok

1
Ha egyenletesen csúszik le, akkor a gyorsulása nulla, vagyis a súrlódás megegyezik a gravitáció lejtő irányú komponensével.
Először ki kell számolni a súrlódási együtthatót:

Rajzoltam ábrát.
A testre azok az erők hatnak, amiket vastag nyilak jelölnek:
- m·g súlyerő
- T tartóerő
- Fs súrlódás
Az F₁ és F₂ erők önmagukban nincsenek, azok csak az m·g komponensei. Viszont azokat kell kiszámolni, mert azok határozzák meg a többit:
- Lejtőirányú komponens: F₁ = m·g · √3/2
- Lejtőre merőleges komponens: F₂ = m·g · 1/2
ugyanis a 60°-os derékszögű háromszög befogói √3/2 és 1/2 hosszúak, ha az átfogó 1. Az ábrán bejelöltem azt a két szöget is, amik pont akkorák, mint a lejtő szöge, szóval 60 fokosak.

Na most a súrlódás lehetséges maximális értékét az határozza meg, hogy mekkora az F₂ lejtőre merőleges komponens (a nyomóerő):
Fs = µ·F₂

Ha nem gyorsul a test, akkor Fs = F₁:
µ · m·g · 1/2 = m·g · √3/2
µ = √3

Hmm, ez baromi nagy szám. Ritka, hogy ilyen nagy a súrlódási tényező (a guminak tud lenni ilyen nagy, mondjuk radírgumira gondolok.) Nem lehet, hogy 30 fokos az a lejtő?

Tovább számolok azért:
Ha v = 5 m/s sebességgel indul felfelé a lejtőn, akkor alul ekkora a mozgási energiája:
E₁ = 1/2 · m · v²
Feljut h magasra. Közben s utat tesz meg:
h/s = √3/2 → s = 2h/√3
Mivel magasabbra jut, helyzeti energiája is lesz:
E₂ = m·g·h
Miközben felfelé megy, Fs erő hat rá, ami munkát végez:
W = Fs · s
W = µ · m·g/2 · s
W = µ · m·g · h/√3
W = √3 · mg · h/√3 = m·g·h
(Érdekes, ugyanakkora, mint a helyzeti energia...)

Energiamegmaradás:
E₁ = W + E₂
1/2 · m · v² = mgh + mgh
v² = 4gh
h = v²/(4g) = 5²/40 = 0,625 m
1